Quantcast
  • Register
PhysicsOverflow is a next-generation academic platform for physicists and astronomers, including a community peer review system and a postgraduate-level discussion forum analogous to MathOverflow.

Welcome to PhysicsOverflow! PhysicsOverflow is an open platform for community peer review and graduate-level Physics discussion.

Please help promote PhysicsOverflow ads elsewhere if you like it.

News

PO is now at the Physics Department of Bielefeld University!

New printer friendly PO pages!

Migration to Bielefeld University was successful!

Please vote for this year's PhysicsOverflow ads!

Please do help out in categorising submissions. Submit a paper to PhysicsOverflow!

... see more

Tools for paper authors

Submit paper
Claim Paper Authorship

Tools for SE users

Search User
Reclaim SE Account
Request Account Merger
Nativise imported posts
Claim post (deleted users)
Import SE post

Users whose questions have been imported from Physics Stack Exchange, Theoretical Physics Stack Exchange, or any other Stack Exchange site are kindly requested to reclaim their account and not to register as a new user.

Public \(\beta\) tools

Report a bug with a feature
Request a new functionality
404 page design
Send feedback

Attributions

(propose a free ad)

Site Statistics

205 submissions , 163 unreviewed
5,047 questions , 2,200 unanswered
5,345 answers , 22,709 comments
1,470 users with positive rep
816 active unimported users
More ...

  Question about Majorana fermions

+ 7 like - 0 dislike
1050 views

I have a few questions about Majorana fermions.

  1. What is Majorana mass? Does it have a different value compared to the mass in the Dirac equation for an arbitrary fermion? How exactly do they differ?

  2. Can the Majorana equation be rewritten in form two-component spinor equation? Or it is two-component already?

This post imported from StackExchange Physics at 2014-03-05 14:50 (UCT), posted by SE-user Andrew McAddams
asked Nov 20, 2013 in Theoretical Physics by Andrew McAddams (340 points) [ no revision ]
retagged Apr 19, 2014 by dimension10
Please clarify: What do you mean by "different value" for the mass? Do you mean observed masses in nature? Furthermore, what do you mean by "two-component at once"?

This post imported from StackExchange Physics at 2014-03-05 14:50 (UCT), posted by SE-user Frederic Brünner
@FredericBrünner : "Do you mean observed masses in nature?", - yes, I do. "...Furthermore, what do you mean by "two-component at once"? ..", - there are two forms of Majorana equation: for real and complex spinor. The second refer to two-component spinor, the first refers for four-component spinor. But generally Majorana fermion has only two components, doesn't it?

This post imported from StackExchange Physics at 2014-03-05 14:50 (UCT), posted by SE-user Andrew McAddams
You might want to break this up into two questions.

This post imported from StackExchange Physics at 2014-03-05 14:50 (UCT), posted by SE-user Dan

1 Answer

+ 7 like - 0 dislike

We know that we can describe a spin $1/2$ massless particle using only a single Weyl field (lets say left-handed $\psi_{L}$). To introduce a mass term we have to use two spinor fields (one left-handed and one right-handed) and this gives the Dirac mass term. The question is now that if we can describe a massive particle with a single Weyl field. Well yes, due to the fact that given a left-handed Weyl spinor, it is possible to construct a right-handed spinor $\psi_{R}=i\sigma^{2}\psi_{L}^{*}$. Thus, we can write the Dirac equation using $i\sigma^{2}\psi_{L}^{*}$

$$\hspace{43mm} \bar{\sigma}^{\mu}i\partial_{\mu}\psi_{L}=im\sigma^{2}\psi_{L}^{*} \hspace{30mm}(1)$$

The known algebraic methods performed for the Dirac equation to prove that it implies a massive Klein-Gordon equation can be performed here without any problems. Thus, the above equation implies $(\Box+m^{2})\psi_{L}=0$. Here we have constructed a mass term using only $\psi_{L}$ and this is known as Majorana mass. The similarity with the Dirac mass can be seen by writting $(1)$ in terms of the four component Majorana spinor $\psi_{m}$ in the chiral representation

$$\psi_{m}=\begin{pmatrix} \psi_{L}\\i\sigma^{2}\psi_{L}^{*} \end{pmatrix}$$

Now, equation $(1)$ becomes

$$(i\gamma_{\mu}\partial^{\mu}-m)\psi_{m}=0$$

The Majorana mass has a very important physical difference when compared to the Dirac mass. We know that the Dirac action with a mass term is invariant under a global $U(1)$ transformations of $\psi_{L}$ and $\psi_{R}$ (i.e. $\psi_{L}\rightarrow e^{i\alpha}\psi_{L},\hspace{2mm}\psi_{R}\rightarrow e^{i\alpha}\psi_{R}$).But for Majorana spinors, $\psi_{L}$ and $\psi_{R}$ are not independent, they are related by complex conjugation. So, if $\psi_{L}$ transforms as $\psi_{L}\rightarrow e^{i\alpha}\psi_{L}$then $\psi_{R}$ transforms like $\psi_{R}\rightarrow e^{-i\alpha}\psi_{R}$. The Majorana equation $(1)$ is not invariant under global $U(1)$ symmetries. This fact implies that a spin $1/2$ particle with a $U(1)$ conserved charge cannot have a Majorana mass. All spin $1/2$ particles with an electric charge cannot have a Majorana mass. Also leptons that have a Majorana mass violate the lepton number (because this is a $U(1)$ symmetry).

One possible particle that could have a Majorana mass is the neutrino. But this is yet to be determined. (I didn't answer your questions point by point but I hope this clarifies some of them).

This post imported from StackExchange Physics at 2014-03-05 14:50 (UCT), posted by SE-user Leonida
answered Nov 21, 2013 by Leonida (130 points) [ no revision ]
Thank you for your answer. It is useful for me. Can you also answer what is relations between expressions for Dirac and Majorana masses (if they may be compared with each other)? By the other words, does exist relation like $$ m_{Dirac} = m_{Majorana} - A, \quad A = const? $$

This post imported from StackExchange Physics at 2014-03-05 14:50 (UCT), posted by SE-user Andrew McAddams
The short answer is no. A relation like that does not exist. But, there are some "connections" for the masses in the See-Saw mechanism. Not knowing much about this mechanism, here are some good (I hope) articles :ias.ac.in/pramana/v72/p217/fulltext.pdf kvi.nl/~loehner/saf_seminar/2010/NeutrinoMassAndNewPhysics.pdf

This post imported from StackExchange Physics at 2014-03-05 14:50 (UCT), posted by SE-user Leonida
I return to your great answer again and want to ask: how to associate lepton number conservation with the $U(1)$-symmetry? This is not a gauge symmetry, so it must be something global. But how to build leptonic current (or, by the other words, charge conservation law) in this case?

This post imported from StackExchange Physics at 2014-03-05 14:50 (UCT), posted by SE-user Andrew McAddams

Your answer

Please use answers only to (at least partly) answer questions. To comment, discuss, or ask for clarification, leave a comment instead.
To mask links under text, please type your text, highlight it, and click the "link" button. You can then enter your link URL.
Please consult the FAQ for as to how to format your post.
This is the answer box; if you want to write a comment instead, please use the 'add comment' button.
Live preview (may slow down editor)   Preview
Your name to display (optional):
Privacy: Your email address will only be used for sending these notifications.
Anti-spam verification:
If you are a human please identify the position of the character covered by the symbol $\varnothing$ in the following word:
p$\hbar$ysicsOve$\varnothing$flow
Then drag the red bullet below over the corresponding character of our banner. When you drop it there, the bullet changes to green (on slow internet connections after a few seconds).
Please complete the anti-spam verification




user contributions licensed under cc by-sa 3.0 with attribution required

Your rights
...